Đến nội dung

JUV nội dung

Có 136 mục bởi JUV (Tìm giới hạn từ 05-05-2020)



Sắp theo                Sắp xếp  

#635035 Xét tất cả các số nguyên dương thỏa mản n là bội của 2003, tìm giá trị bé nhấ...

Đã gửi bởi JUV on 23-05-2016 - 20:23 trong Số học

Spoiler

Vì số nguyên dương $n$ là bội của $2003$ nên $n$ nhỏ nhất là 2003 có tổng các chữ số là $5$

Ta sẽ chứng minh $MinS_{n}=5$.Thật vậy,giả sử tồn tại $S(n)<5$

Vì $S(n)<5$ và $n$ là số nguyên dương đồng thời là bội của $2003$ nên $S(n)\epsilon \left [ 1;4 \right ]$

Mặt khác không tồn tại số nguyên dương nào có tổng các chữ số là 1 mà lại chia hết cho 2003 

Vậy $S(n)\epsilon \left [ 2;4 \right ]$ 

Suy ra chữ số tận cùng của bội 2003 lần lượt là $1,2,3$

+Nếu bội của 2003 có tận cùng là $1$ thì tồn tại 1 số $\overline{P7}$ sao cho $2003.\overline{P7}$ có $S(n)<5$.

$2003.\overline{P7}=20030.P+2003.7\Rightarrow P<2$.Thử các giá trị thì nhận thấy $S(n)$ trong trường hợp này luôn lớn hơn $5$ ($P$ là số tự nhiên)

+Nếu bội của 2003 có tận cùng là $2$ thì tồn tại 1 số $\overline{P4}$ sao cho $2003.\overline{P4}$ có $S(n)<5$.

$2003.\overline{P4}=20030.P+2003.4\Rightarrow P< 2$.Thử các giá trị thì nhận thấy $S(n)$ trong trường hợp này luôn lớn hơn $5$ ($P$ là số tự nhiên)

+Nếu bội của 2003 có tận cùng là $3$ thì tồn tại 1 số $\overline{P1}$ sao cho $2003.\overline{P1}$ có $S(n)<5$.

$2003.\overline{P1}=20030.P+2003.1\Rightarrow P<2$.Thử các giá trị thì nhận thấy $S(n)$ trong trường hợp này luôn lớn hơn $5$ ($P$ nguyên dương)

Vậy $MinS_{n}=5$ khi $n=2003$

Cách làm của em có vẻ không đúng lắm vì việc $P<2$ không phải điều kiện để $S_n$ nhỏ hơn 5 vì không phải $n$ càng nhỏ thì $S_n$ càng nhỏ.




#635332 Xét tất cả các số nguyên dương thỏa mản n là bội của 2003, tìm giá trị bé nhấ...

Đã gửi bởi JUV on 24-05-2016 - 23:36 trong Số học

Em nghĩ là khi $P<2$ mà $P$ là số tự nhiên thì $P=0$ hoặc $P=1$ thì khi thay vào cái $2003.\overline{P7}$ và những cái tương tự thì khi tính $S(n)$ luôn có $S(n)>5$

P/s:Em cũng thấy lời giải của em nó có phần gượng gạo,anh có ý tưởng hay lời giải nào tốt hơn thì cho tụi em xem với ạ :D

Bài này nếu giải ra 1 cách đầy đủ thì cực kì rối, phải sử dụng nhiều đến công cụ máy tính và 1 vài định lí số học của THPT. Đáp số đúng là $S_n=3$, số thoả mãn thì cực to và với $S_n$ "đặc biệt" như vậy thì cũng khó mà lập luận logic để mà chứng minh được




#678245 USAMO 2017 ngày 1

Đã gửi bởi JUV on 21-04-2017 - 20:55 trong Thi HSG Quốc gia và Quốc tế

Bài 2:

Ta sẽ chứng minh rằng số hoán vị chứa đúng $k$ $A-inversion$ không phụ thuộc vào dãy $A$ (Và bằng số số hoán vị có đúng $k$ nghịch thế).

Bài toán hiển nhiên đúng với $n=1$, giả sử bài toán đúng với $n=k$, xét $n=k+1$. Xét $1$ hoán vị có chứa số $w_1$ đứng đầu dãy. Gọi $f$ là số số $w_i$ thoả mãn $(w_1;w_j)$ là $1$ $A-inversion$. Nếu $a_1>w_1$, số số $w_j$ thoả mãn điều kiện bằng số số $t$ trong $n$ số $m_1,m_2,...,m_n$ thoả mãn $t>a_1$ hoặc $t<w_1$, số các số đó hoàn toàn không phụ thuộc vào cách sắp xếp $n-1$ số cồn lại khác $w_1$. Tương tự với trường hợp $a_1<w_1;a_1=w_1$. Gọi số số $w_j$ lạp thành cặp $A-inversion$ với $w_1$ là $f$, ta đếm cách sắp xếp $n-1$ số còn lại để có đúng $k-f$ cặp, và nó không phụ thuộc vào $A-a_1$. Tương tự với các trường hợp những số khác xếp ở vị trí đầu trong hoán vị, tổng lại các cách sắp xếp thì ta được số cách sắp xếp để có đúng $k$ cặp $A-inversion$ và nó không phụ thuộc vào $A$. Mệnh đề đúng với $n=k+1$, $\Rightarrow$ $Q.E.D$




#635570 USAMO 2016

Đã gửi bởi JUV on 25-05-2016 - 21:42 trong Thi HSG Quốc gia và Quốc tế

 

Bài 6. Cho $n, k$ là hai số nguyên, $n \ge k \ge 2$. Bạn tham gia vào một trò chơi với một phù thuỷ ác độc (?!)
Phù thuỷ có $2n$ lá bài, với mỗi $i = 1, 2, \cdots , n$, thì có đúng hai lá bài cùng được đánh số là $i$. Lúc đầu, phù thuỷ sắp úp tất cả lá bài trên một hàng, thứ tự bất kỳ.
Về phần bạn, bạn sẽ lặp lại các thao tác sau: chỉ vào $k$ lá bài bạn muốn chọn, phù thuỷ sẽ lật ngửa lên hộ bạn. Nếu lật lên, có hai tấm thẻ cùng được đánh 1 số, bạn thắng, game sẽ kết thúc. Nếu ngược lại, phù thuỷ sẽ hoán vị $k$ lá bài đó và lật úp chúng lại. Mỗi lần bạn làm vậy là một bước đi. Và bạn sẽ lặp lại thao tác trên.
Ta gọi game này là thành công nếu tồn tại $m$ nguyên dương nào đó và một chiến thuật nào đó ở tối đa $m$ bước đi, không quan trọng việc phù thuỷ xáo trộn bài của bạn.
Hỏi với $n$ và $k$ nào thì game này thành công?

 

Ta sẽ chứng minh rằng $2\leq k< n$ là điều kiện cần và đủ để ta có thể chiến thắng.

Đầu tiên xét $2\leq k< n$, ta sẽ đặt $2n$ tấm bài vào $2n$ hộp được đánh số từ 1 đến $2n$. Ta sẽ chọn ra $k$ lá bài trong $k$ hộp để xem bài, sau đó phù thuỷ sẽ hoán vị $k$ lá bài đó vào $k$ hộp mà ban đầu đựng chúng.Ta sẽ giả sử không có may mắn trong các lần chọn và ta không thể chọn được 2 lá bài ghi cùng số trước khi có thể đến được bước mà ta có thể chắc chắn chiến thắng. Đầu tiên ta sẽ xem các lá bài ở trong $k$ hộp $1,2,...,k$. Ta sẽ ghi lại $k$ số này trên giấy và phù thuỷ sẽ hoán đổi các lá bài trong $k$ hộp này. Sau đó ta sẽ lật $k$ lá bài trong $k$ hộp $2,3,...,k+1$ và ghi $k$ số này trên giấy. Vì $k-1$ lá bài trong các hộp $2,3,...,k$ đã được ghi trên giấy ở lần đầu nên trong 2 lần ghi số trên giấy thì có ít nhất  $k-1$ số được ghi 2 lần.Nếu trong 2 lần chỉ có đúng $k-1$ số được ghi lại 2 lần thì sẽ có duy nhất 1 số được ghi ở lần 1 mà không được ghi ở lần 2, gọi số đó là $x$.Số của lá bài nằm ở hộp 1 sau lần lật bài thứ nhất đã được ghi 1 lần nhưng đến lần thứ 2 thì nó sẽ không được ghi, vì vậy lá bài ở trong hộp 1 hiện tại có số $x$. Nếu trong cả 2 lần lật bài mà cả $k$ số được ghi lại ở cả 2 lần thì ta sẽ gọi số của lá bài trong hộp $k+1$ trong lần lật bài thứ 2 là $y$. Theo giả sử vì ta không có may mắn nên $k-1$ số ghi trong các lá bài trong $k-1$ hộp $2,3,...,k$ trong lần lật bài thứ 2 đều khác $y$. Vì vậy lá bài ở trong hộp 1 hiện tại chứa số $y$. Vì vậy trong cả 2 trường hợp thì ta đều có thể xác định số của lá bài trong hộp 1 sau 2 lần lật bài. Trong lần lật bài thứ $p$ với $p\leq 2n-k+1$, ta sẽ lật bài ở trong các hộp $p,p+1,...,p+k-1$. Sau lần lật thứ 2, ta có thể xác định số quân bài trong hộp 1 và cứ tương tự như thế, sau lần lật bài thứ $p$, ta sẽ xác định được số quân bài trong hộp $p-1$ với $p\leq 2n-k+1$. Vì vây sau lần lật bài thứ $2n-k+1$ thì ta sẽ xác định được $2n-k$ số của các lá bài trong hộp $1,2,...,2n-k$ và vị trí của chúng trong $2n-k$ hộp đó. Có $k<n$ nên $2n-k\geq n+1$. Vì vậy trong $2n-k$ hộp đã biết thì luôn có 2 hộp có lá bài mang cùng số. Ta sẽ chọn lật 2 lá bài đấy và chiến thắng

Với $n=k$, ta sẽ chứng minh được rằng không thể chiến thắng nếu không có may mắn . Ban đầu phù thuỷ sẽ sơn tất cả mặt dưới các lá bài màu trắng và úp chúng xuống. Sau đó phù thuỷ sẽ phù phép sao cho khi ta chọn $n$ lá bài để lật lên thì mặt dưới của $n$ lá bài đó sẽ chuyển thành 1 màu khác với tất cả các màu của $n$ lá bài còn lại. Giả sử người chơi có 1 cách để chắc thắng mà không cần may mắn. Để đẩy người chơi vào cảnh thua cuộc thì ta có thể thêm 1 luật như sau:

Sau khi người chơi chọn n hộp có chứa n lá bài để lật thì phù thuỷ có quyền được hoán đổi các lá bài có cùng màu với nhau vào các hộp của chúng một cách nào đó rồi sau đó màu ở mặt dưới n lá bài được người chơi chọn sẽ đổi màu thành 1 màu mà không trùng với bất cứ lá bài nào không được chọn và sau đó mới được lật lên cho người chơi xem

Việc này không ảnh hưởng đến kế hoạch chiến thắng của người chơi vì ở mỗi lần lật bài, cho dù phù thuỷ có đổi vị trí thế nào thì đó cũng chỉ là 1 trường hợp con của tập các trường hợp mà trò chơi có thể diễn ra trong đầu người chơi. Ta gọi 1 màu trong số các màu trên các quân bài sau lần lật bài thứ $p$ là màu đại diện nếu như sau lần lật bài thứ $p$, $n$ lá bài mà người chơi chọn chuyển thành màu đó. Ta thấy rằng tại mọi thời điểm thì có đúng $n$ lá bài có màu đại diện, $n$ lá bài còn lại mang nhiều nhất $n$ màu khác nhau nên tại mọi thời điểm, $2n$ lá bài được tô bằng nhiều nhất $n+1$ màu. Với mọi số tự nhiên $t$, trước lần lật bài thứ $t$, phù thuỷ có thể hoán vị các quân bài cùng màu để khi người chơi lật $n$ quân bài mà anh ta chọn, $n$ quân bài đó có ghi $n$ số là $1,2,3,...,n$. Vì đầu tiên tất cả các quân bài mang màu trắng nên phù thuỷ có thể hoán đổi vị trí $2n$ quân bài sao cho người chơi không thể chiến thắng. Mệnh đề đúng với $t=1$, giả sử mệnh đề đúng với $t=k$, xét lần lật bài thứ $k+1$. Sau lần lật bài thứ $k$ thì người chơi lật $n$ lá bài mang các số từ 1 dến $n$ nên trước lần lật bài thứ $k+1$, các lá bài mang màu đại diện được có các số từ 1 đến $n$. Vì mỗi số chỉ xuất hiện 2 lần trên đúng 2 lá bài và các lá bài mang màu đại diện chứa các số từ 1 đến $n$ nên không có 2 lá bài nào không mang màu đại diện mà số ghi trên 2 lá bài đó trùng nhau. Ở trong lần lật bài thứ $k+1$, người chơi sẽ chọn 1 số lá bài mang màu đại diện và 1 số lá bài không mang màu đại diện. Giả sử người chơi chọn $i$ lá bài mang màu đại diện và $n-i$ lá bài không mang màu đại diện thì $n-i$ lá bài không mang màu đại diện đó có chứa $n-i$ số khác nhau trên mỗi lá bài trong $n$ số $1,2,...,n$ theo chứng minh trên. Mà $n$ lá bài mang màu đại diện có ghi $n$ số phân biệt $1,2,...,n$ nên có thể hoán đổi vị trí các lá bài đó sao cho với $i$ lá bài mang màu đại diện được chọn thì sẽ có $i$ giá trị khác nhau trong tập $1,2,...,n$ mà mỗi số trong $i$ số đó không được viết lên bất cứ $n-i$ lá bài được chọn không mang màu đại diện. Vì vậy sau khi lật bài, người chơi sẽ thấy các số $1,2,...,n$ được viết trên $n$ lá bài. Vì vậy dù chọn thế nào thì người chơi không thể chọn được 2 lá bài mang cùng số. Vì vậy người chơi không thể chiến thắng.

Vậy điều kiện của $n$ và $k$ là: $n\geq 3$, $2\leq k\leq n-1$




#643666 USA TSTST 2016

Đã gửi bởi JUV on 04-07-2016 - 21:05 trong Thi HSG Quốc gia và Quốc tế

Bài 5: Giả sử xe ủi có thể đi được như đề bài yêu cầu. Ta đặt $1$ con kiến ở $1$ đầu mút của $1$ bức tường có hoành độ nhỏ hơn hoặc bằng bất kì đầu mút nào của các bức tường khác. Sau đó con kiến sẽ di chuyển ngang theo chiều dương của trục Ox theo quy tắc sau:

-Khi con kiến đang di chuyển dọc theo $1$ bức tường thì nếu đến lúc nào đó khi phía trên nó là $1$ đầu mút của bức tường khác thì nó sẽ nhảy lên đầu mút đó rồi di chuyển tiếp( trường hợp có nhiều đầu mút thì chọn đầu mút có tung độ lớn nhất)

-Nếu con kiến đi từ đầu đến hết một bức tường mà tại mọi thời điểm không có đầu mút của bức tường nào phía trên nó thì nó sẽ dừng di chuyển.

Ta thấy rằng đến $1$ lúc nào đó con kiến sẽ dừng di chuyển. Lúc đó có $1$ bức tường mà $1$ đầu mút của nó là điểm mà con kiến dừng lại. Khi đó bất kì điểm nào nằm vào phía trên của bức tường đó và hoành độ nằm vào khoảng giữa $2$ hoành độ của $2$ đầu mút bức tường thì điểm đó sẽ không nằm vào bất kì bức tường nào. Vì vậy để xe ủi có thể đi vào mặt trên bức tường đó thì xe phải đi đến mặt đó từ phía ngang. Vì vậy khi đâm vào mặt đó thì xe ủi sẽ đi lên trên và không đâm vào bất cứ bức tường nào nữa. Vì vậy sau khi tông vào cả $2$ mặt của tất cả bức tường, xe ủi sẽ đi lên trên. Chứng minh tương tự thì sau khi tông vào tất cả các bức tường, xe ủi sẽ đi xuống dưới, suy ra mâu thuẫn. Từ đó suy ra dpcm




#633604 Tìm và chứng minh các giá trị có thể có của đường chéo của bảng vuông

Đã gửi bởi JUV on 17-05-2016 - 00:53 trong Tổ hợp và rời rạc

Đánh số các hàng từ trên xuống dưới lần lượt là $1,2,...,n$, đánh số các cột từ trái sang phải lần lượt là $1,2,...,n$. Gọi số điền trong ô $(1;1)$ là $x$, xét tập $A=\left \{r_1,r_2,...,r_n \right \}$ sao cho số được viết trong ô $(1;i)$ là $r_i+x$. Tập $B=\left \{q_1,q_2,...,q_n \right \}$ sao cho ô $(i;1)$ được viết số $q_i+x$. Theo đề bài, dễ chứng minh được ô $(i;j)$ được viết số $x+q_i+r_j$. Vì vậy tổng các số trên bảng bằng $\sum_{q_i\in B;r_j\in A}^{}$$x+q_i+r_j$=$\sum_{i=1}^{n^2}i$=$\frac{n(n+1)(2n+1)}{6}$

$\rightarrow$ $n^2x+n\sum_{i=1}^{n}r_i +n\sum_{i=1}^{n}q_i$=$\frac{n(n+1)(2n+1)}{6}$

$\rightarrow$ $\sum_{i=1}^{n} x+r_i+q_i$=$\frac{(n+1)(2n+1)}{6}$, Dễ thấy tổng đó là tổng các số trên các ô nằm trên đường chéo chính

Nhân tiện SL Challenge Competition là Sri Lanka Challenge Competition




#647662 Tìm tất cả dạng có thể của số trên bảng

Đã gửi bởi JUV on 02-08-2016 - 19:36 trong Tổ hợp và rời rạc

Bài 8: Đặt ước lẻ lớn nhất của $b-a$ là $t$, ta sẽ chứng minh rằng $\forall k\in \mathbb{N}$ ta có thể viết lên bảng cặp số $(k;k+xt)$ với $x$ là số nguyên dương bất kì. Dễ thấy rằng với mọi cặp $(m;n)$ được viết trên bảng, đề có $m<n$ và $n-m \vdots t$. Từ cặp $(a;b)$ ban đầu, đặt $p=b-a=2^{i}t$, ta có thể tạo ra được cặp có dạng $(2^c;2^c+p),(2^c+p;2^c+2p);...;(2^c+2^{c-i}p-p;2^c+2^{c-i}p)$ với $c>i$. Vậy ta có thể tạo ra cặp $(2^c;2^c+2^ct)$, từ đó ta có thể tạo cặp $(1;t+1)$, từ cặp này dễ tạo ra được các cặp khác thỏa mãn giả thiết

Bài 7: Mình đã chứng minh ở đây: http://diendantoanho...-ít-nhất-n-lần/




#654511 Tìm tất cả các tập hợp $A$ gồm hữu hạn các số thực không âm khác nhau

Đã gửi bởi JUV on 17-09-2016 - 18:40 trong Tổ hợp và rời rạc

Giả sử $A$ có $n$ phần tử là $a_1<a_2<...<a_n$, xét các tổng $f(x;y;z;t)=a_xa_y+a_za_t$. Theo định nghĩa tập $A$ thì các số sau đều thuộc $A$: $f(1;3;2;4)<f(1;2;3;4)<f(1;2;3;5)<f(1;2;3;6)<...<f(1;2;3;n)<f(1;2;4;n)<f(1;2;5;n)<...<f(1;2;n-1;n)<...<f(n-3;n-2;n-1;n)$, tổng cộng $4n-14$ số. Vì vậy ta có $n\geq 4n-14$ hay $n\leq 4$. Vậy $n=4$, đến đây dễ thấy các tập $A$ thoả mãn là $\left \{ a,b,c,d\mid (a-b)(b-c)(c-d)(d-a)(a-c)(b-d)\neq 0; b\neq 1; a=\frac{-cd}{b-1}; a\geq 0;b\geq 0;c\geq 0;d\geq 0 \right \}$




#642029 Tìm tất cả các số nguyên dương n và k thỏa mãn yêu cầu bài toán

Đã gửi bởi JUV on 24-06-2016 - 18:56 trong Tổ hợp và rời rạc

Xét 1 người $A$ bất kì, người đó sẽ giải 3 bài toán bất kì, mỗi bài toán sẽ có $k$ người giải nó tính cả $A$. Vì mỗi người trong số $n$ người với $A$ có đúng 1 bài toán mà 2 người đều giải được nên trong 3 bài toán mà $A$ giải được thì không có người nào mà giải được ít nhất 2 trong số 3 bài. Nhưng mỗi người trong số $n-1$ người còn lại giải được ít nhất $1$ trong số $3$ bài toán đó nên tồn tại 1 song ánh giữa tập $n-1$ người đã trừ $A$ với tập các người giải được ít nhất $1$ bài toán trong số $3$ bài đã nêu trừ $A$. Vì vậy $n=3(k-1)+1$. Vì mỗi người giải được $3$ bài toán và mỗi bài toán được giải bởi $k$ người nên số bài toán được giải là $\frac{3n}{k}$, vì vậy $k$ là ước của $3n$ hay là ước của $9(k-1)+3$. Vì vậy $k$ là ước của $6$. Tuy nhiên nếu xét $k=6$ thì giả sử người $A$ làm được $3$ bài $1,2,3$ thì trong số $15$ người còn lại có $5$ người làm được bài $1$, $5$ người làm được bài $2$,$5$ người làm được bài $3$. Xét những người làm được bài $4$ thì theo nguyên lí Pigeonhole thì tồn tại $2$ người cùng làm được bài $4$, $2$ người cùng làm được bài $1$ hoặc $2$ hoặc $3$ (vô lí vì $2$ người bất kì thì chỉ làm được 1 bài chung). Vì vậy $(k;n)=(1;1);(2;4);(3:7)$




#691756 Tuần 5 tháng 8/2017: $YC=ZB$

Đã gửi bởi JUV on 28-08-2017 - 23:01 trong Chuyên mục Mỗi tuần một bài toán Hình học

Bài 1:  Gọi $O_1$,$O_2$ là tâm đường tròn $CMZ$ và $BNY$. Dễ thấy $\angle CO_1Z=\angle BO_2Y=\alpha = \pi -\angle A$ (do $\angle CMZ=\angle BNY= \frac{\pi+\angle A}{2}$) và $CO_1Z$ và $BO_2Y$ cân tại $O_1,O_2$ nên $\exists k\in R$ sao cho $O_1\in f_1(AB), O_2\in f_2(AC)$ với $f_1$ là phép đồng dạng quay với tâm $C$, hệ số đồng dạng $k$, hệ số góc $\beta $ và $f_2$ là phép đồng dạng quay tâm $B$, hệ số đồng dạng $k$, hệ số góc $-\beta$ với $\beta=\frac{\pi -\alpha }{2}$. Dễ thấy $f_1(AB)=f_2(AC)=AD$ và $f_1(B)=f_2(C)=I$ là điểm chính giữa cung $BC$ không chứa $A$ của $(ABC)$ . Gọi $K_1$,$K_2$ là trung điểm $DC$,$DB$. Ta có $O_1M_1$ đi qua trung điểm $CE$ (do $O_1M_1$ là trung trực $MC$), tương tự $O_2M_2$ đi qua trung điểm $BF$ và $O_1,O_2$ đều thuộc $AD$ nên theo định lý $Menelaus$, $O_1\equiv O_2\equiv O$. Vậy $BZ=\frac{OI}{k}=CY$




#691712 Tuần 5 tháng 8/2017: $YC=ZB$

Đã gửi bởi JUV on 27-08-2017 - 22:13 trong Chuyên mục Mỗi tuần một bài toán Hình học

Như vậy lời giải cho hai bài Tuần 4 tháng 8/2017 đã được đưa tại đây kèm theo đó là hai bài toán mới của thầy Trần Quang Hùng và bạn Đỗ Xuân Long. Xin được trích dẫn lại hai bài toán:

Bài 1:  Cho tam giác $ABC$ có phân giác $AD$.$E,F$ lần lượt thuộc $CA,AB$ sao cho $EF$ song song $BC$. Gọi $M,N$ theo thứ tự là hình chiếu của $C,B$ lên $DE,DF$. $AD$ cắt đường tròn ngoại tiếp các tam giác $AEM$ và $AFN$ tại lần lượt tại $U,V$ khác $A$. Gọi $NV,MU$ lần lượt cắt $CA,AB$ tại $Y,Z$ Chứng minh rằng $YC=ZB$

Hình vẽ:

etXvnkZ.png

Bài 2: Cho lục giác $ABCDE$ nội tiếp có $AB=CD=EF$ và $BC=DE$. $P$ là một điểm di chuyển trên cung nhỏ $AF$ của đường tròn ngoại tiếp lục giác. $PC,PD$ lần lượt cắt $AE,FB$ tại $M,N$. $K,L$ theo thứ tự thuộc các cạnh $AB,EF$ sao cho $MK,NL,AF$ đôi một song song. $PC,PD$ lần lượt cắt $AF$ tại $S,T$. $KS$ cắt $LT$ tại $Q$. Chứng minh rằng đường thẳng $PQ$ chia đôi đoạn $CD$

Hình vẽ

nFt2DRJ.png




#690469 Tuần 3 tháng 8/2017: $PQ$ chia đôi $CD$

Đã gửi bởi JUV on 13-08-2017 - 21:58 trong Chuyên mục Mỗi tuần một bài toán Hình học

Như vậy lời giải cho hai bài Tuần 2 tháng 8/2017 đã được đưa tại đây kèm theo đó là hai bài toán mới của thầy Trần Quang Hùng và bạn Đỗ Xuân Long. Xin được trích dẫn lại hai bài toán:

Bài 1: Cho lục giác $ABCDE$ nội tiếp có $AB=CD=EF$ và $BC=DE$.$P$ di chuyển trên cung nhỏ $AF$ của đường tròn ngoại tiếp lục giác. $PC,PD$ lần lượt cắt $AE,BF$ tại $M,N$.$K,L$ theo thứ tự là hình chiếu của $M,N$ lên cạnh $AF$. $ML$ cắt $NK$ tại $Q$. Chứng minh đường thẳng $PQ$ chia đôi $CD$

Hình vẽ:

ko7ppyI.png
Bài 2: Cho tam giác $ABC$ nhọn, $1$ đường tròn $(K)$ đi qua $B,C$ lần lượt cắt $CA,AB$ tại $E,F$. $(I),(J)$ tiếp xúc $AR$ tại $R$ và tiếp xúc trong với $(K)$ theo thứ tự tại $M,N$ sao cho $I,J$ đều nằm trong các góc 
$\angle FRB$ và $\angle ERC$. Chứng minh $ME,NF$ cắt nhau trên đường thẳng $AR$

Hình vẽ: 

fhLYh0E.png




#689100 Tuần 1 tháng 8/2017: $AU,BV,CW$ đồng quy

Đã gửi bởi JUV on 30-07-2017 - 19:45 trong Chuyên mục Mỗi tuần một bài toán Hình học

Như vậy lời giải cho hai bài Tuần 3 tháng 7/2017 đã được đưa tại đây kèm theo đó là hai bài toán mới của thầy Trần Quang Hùng và thầy Nguyễn Minh Hà. Xin được trích dẫn lại hai bài toán:

Bài 1: (Thầy Trần Quang Hùng) Cho tam giác $ABC$ có điểm $Lemoine$ $L$.$X,Y,Z$ lần lượt nằm trên $LA,LB,LC$ sao cho $YZ,ZX,XY$ lần lượt song song với $BC,CA,AB$. $BZ$ cắt $CY$ tại $D$, $CX$ cắt $AZ$ tại $E$, $AY$ căt $BX$ tại $F$. $U,V,W$ lần lượt đẳng giác với $D,E,F$ trong $LBC,LCA,LAB$. Chứng minh $AU,BV,CW$ đồng quy.

Hình vẽ:  

SUp0Qp8.png

Bài 2: (Thầy Nguyễn Minh Hà) Cho tam giác $ABC$ không đều, $L$ là điểm $Lemoine$. Đường đối trung từ $L$ của $LBC,LCA,LAB$ theo thứ tự cắt lại $(LBC),(LCA),(LAB)$ tại $D,E,F$, Chứng minh $AD,BE,CF$ đồng quy tại $1$ điểm thuộc đường thẳng $Euler$ của $ABC$

Hình vẽ : IjWwPqP.png




#693654 Tuần $4$ tháng $9/2017$: $AP$ đi qua điểm cố định

Đã gửi bởi JUV on 24-09-2017 - 19:45 trong Chuyên mục Mỗi tuần một bài toán Hình học

Như vậy lời giải cho hai bài Tuần 3 tháng 9/2017 đã được đưa tại đây kèm theo đó là hai bài toán mới của thầy Trần Quang Hùng và thầy Nguyễn Tiến Dũng. Xin được trích dẫn lại hai bài toán:

Bài 1: Cho tam giác $ABC$ nội tiếp đường tròn $(O)$ với $B,C$ cố định và $A$ thay đổi trên $(O)$. $I$ là tâm nội tiếp, $BI,CI$ cắt lại $(O)$ tại $E,F$. Lấy $M,N$ để $AM,EM,AN,FN$ lần lượt vuông góc $AF,CF,AE,BE$. Đường qua trung điểm $AI$ song song $BC$ cắt $AB,AC$ tại $R,Q$. $K,L$ là hình chiếu của $A$ lên $FM,EN$. $QL$ cắt $RK$ tại $P$. Chứng minh $AP$ luôn đi qua điểm cố định khi $A$ thay đổi

Hình vẽ
B3tAIUU.png
Bài 2: Cho $ABC$ nội tiếp đường tròn $(O)$ có $2$ điểm Brocard $\Omega_1, \Omega_2$. Chứng minh nếu có $1$ trong $6$ góc $\angle A\Omega_1 O,\angle B\Omega_1O, \angle C\Omega_1O,A\Omega_2 O,\angle B\Omega_2O, \angle C\Omega_2O$ vuông thì có đúng $2$ trong $6$ góc là vuông
Hình vẽ:
4i3Hy4z.png



#694374 Tuần $2/10$ năm $2017$: Tâm $(PBC)$ nằm trên...

Đã gửi bởi JUV on 08-10-2017 - 18:38 trong Chuyên mục Mỗi tuần một bài toán Hình học

Như vậy lời giải cho hai bài Tuần 1, tháng 10/2017 đã được đưa tại đây kèm theo đó là hai bài toán mới của thầy Trần Quang Hùng và thầy Nguyễn Tiến Dũng. Xin được trích dẫn lại hai bài toán:

Bài 1:  Cho tam giác $ABC$ có tâm nội tiếp $I$, phân giác $AD$. $K,L$ là tâm nội tiếp $ABD,ACD$.$J$ là tâm $(AKL)$.$IJ$ cắt $(IKL)$ tại $P$ khác $I$.Chứng minh tâm $(PBC)$ nằm trên $(O)$

Hình vẽ:

D1IjxvU.png

Bài 2: Cho tam giác $ABC$ nội tiếp $(O)$. $D,E$ thuộc $CA,AB$ sao cho $O$ là trung điểm $DE$ và $DE=OA$.$K$ đối xứng $O$ qua $BC$. Lấy $M,N$ để $OM,ON$ lần lượt song song $CA,AB$, $K$ là trung điểm $MN$. $BN$ cắt $CM$ tạp $P$. Chứng minh $(PMN)$ tiếp xúc $(O)$

Hình vẽ:

uM3GYxt.png

 

 



#692818 Tuần $2$ tháng $9/2017$: Chứng minh $\frac...

Đã gửi bởi JUV on 10-09-2017 - 22:33 trong Chuyên mục Mỗi tuần một bài toán Hình học

Như vậy lời giải cho hai bài Tuần 1, tháng 9, 2017 đã được đưa tại đây kèm theo đó là hai bài toán mới của thầy Trần Quang Hùng và anh Ngô Quang Dương. Xin được trích dẫn lại hai bài toán:

Bài 1: Cho tam giác $ABC$ và $M,N$ nằm trên cạnh $BC$ sao cho $M$ nằm giữa $N,B$.Lấy $P,Q$ trên $AM,AN$ để $BP,CQ$ cùng vuông góc với $BC$. $K,J$ là tâm ngoại tiếp $(APQ),(AMN)$. $L$ là hình chiếu của $K$ lên $AJ$. Chứng minh $\frac{AJ}{AL}=\frac{MN}{BC}$

Hình vẽ

eM2iXER.png

Bài 2: Cho tam giác $ABC$ và $l$ là 1 đường thẳng bất kì. $D,E,F$ lần lượt là hình chiếu của $A,B,C$ lên $l$.$X,Y,Z$ lần lượt chia $AD,BE,CF$ theo cùng $1$ tỉ số $k$. Các đường lần lượt qua $X,Y,Z$ và vuông góc $BC,CA,AB$ đồng quy tại $K$. Chứng minh $(KAX),(KBY),(KCZ)$ đồng trục và trục đẳng phương của chúng đi qua điểm cố định khi $k$ thay đổi.

Hình vẽ

z6aGTL9.png




#645640 TRƯỜNG HÈ TOÁN HỌC MIỀN BẮC 2016

Đã gửi bởi JUV on 20-07-2016 - 13:10 trong Thi HSG cấp Tỉnh, Thành phố. Olympic 30-4. Đề thi và kiểm tra đội tuyển các cấp.

Bài 14: 

Dễ chứng minh rằng không có $2$ số nào khác tính chẵn lẻ (Nếu không thì sẽ bỏ được $1$ số sao cho tổng $2009$ số còn lại lẻ). Đương nhiên tất cả các số không thể đều lẻ nên tất cả các số đều chẵn. Chia mỗi số cho $2$ ta được $2010$ số mới thỏa mãn đề bài. Đến đây dùng nguyên lí cực hạn dễ thấy điều vô lí




#646336 TRƯỜNG HÈ TOÁN HỌC MIỀN BẮC 2016

Đã gửi bởi JUV on 24-07-2016 - 21:45 trong Thi HSG cấp Tỉnh, Thành phố. Olympic 30-4. Đề thi và kiểm tra đội tuyển các cấp.

Bài 15 : (Bạn này ghi thiếu một bài mình bổ sung ) Tập số tự nhiên được phân hoạch thành hai tập $A,B$ . Chứng minh rằng với mọi số tự nhiên $n$ tồn tại $a,b>n$ mà $a,b,a+b$ cùng thuộc một tập . 

Ghi chú : Đây là trường hợp bé nhất của định lý Folkman , một trường hợp tiếp theo của nó là : Giả sử tập số tự nhiên được phân hoạch thành ba tập $A,B,C$ . Chứng minh rằng tồn tại $a,b,c$ tự nhiên mà $a,b,c,a+b,a+c,b+c,a+b+c$ cùng thuộc một tập ( là bài toán trong đề thi đội tuyển Anh theo bộ phim A brilliant young mind ) . 

Định lý tổng quát : đây

Với trường hợp $n=0$ thì xét các số từ $1$ đến $6$ sẽ có điều phải chứng minh. Còn với các trường hợp $n$ lớn hơn $0$, có thể đưa về trường hợp $n=0$ bằng cách chỉ xét các số chia hết cho $n+1$ nằm trong $2$ tập hợp. Chia mỗi số đó cho $n+1$ rồi chứng minh tương tự trường hợp $n=0$ 




#643022 Trên bàn cờ 8x8, ta đặt các quân xe màu xanh, đỏ,vàng sao cho các quân xe khá...

Đã gửi bởi JUV on 30-06-2016 - 21:58 trong Toán rời rạc

Giả sử có thể đặt ít nhất $k$ quân cờ mỗi loại lên bàn cờ, ta thấy rằng mỗi cột trong số $8$ cột chỉ chứa ít nhất 1 số loại quân xe cùng $1$ màu, tương tự với mỗi cột. Ta đánh dấu mỗi hàng và mỗi cột theo quy tắc sau:  Với mỗi hàng chỉ chứa quân xe màu vàng thì ta gọi hàng đó là hàng xanh, tương tự màu đỏ là cột đỏ, màu vàng là cột vàng. Đánh số tương tự với mỗi cột, các hàng hoặc cột không có quân xe thì không gọi tên. Gọi số cột xanh, cột đỏ, cột vàng lần lượt là $a_1;a_2;a_3$, số hàng xanh, hàng đỏ, hàng vàng là $b_1;b_2;b_3$. Ta có $a_1+a_2+a_3\leq n$; $b_1+b_2+b_3\leq n$ và ta thấy rằng $1$ quân xe không thể thuộc $1$ hàng hoặc $1$ cột khác màu với nó nên mỗi quân xe chỉ được nằm trên ô thuộc vào cột và hàng cùng màu với quân xe đó. Vì vậy các quân xe xanh chỉ được phép nằm trên $a_1$ cột xanh và trên $b_1$ hàng xanh, tổng cộng mỗi quân xe xanh chỉ được phép nằm trên $1$ trong $a_1b_1$ ô, tương tự mỗi quân đỏ chỉ được nằm trong $1$ trong số $a_2b_2$ ô, mỗi quân vàng chỉ được nằm trong $1$ trong số $a_3b_3$ ô. Nhưng các quân xe mỗi loại có đúng $k$ quân xe nên: $k\leq min(a_1b_1;a_2b_2;a_3b_3)\leq \left [\sqrt[3]{a_1b_1a_2b_2a_3b_3}  \right ]\leq \sqrt[3]{\left [ \frac{a_1+a_2+a_3}{3} \right ]^3\left [ \frac{b_1+b_2+b_3}{3} \right ]^3}\leq \left [ \frac{n+1}{3} \right ]^2$. Vậy giá trị lớn nhất số các quân xe trên bàn cờ là $3\left [ \frac{n+1}{3} \right ]^2$




#647255 Trên 1 ô vuông n x n giả sử điền vào n-1 ô số 1 còn lại điền số 0.

Đã gửi bởi JUV on 30-07-2016 - 22:08 trong Tổ hợp và rời rạc

Ban đầu luôn có hai ô $0$ và $1$ có hiệu là $1$ . Phép biến đổi này ta xét hai TH . Nếu có hai ô $(a,a+1)$ ở đâu đó cạnh nhau mà ta biến đổi ở các vị trí khác hàng và cột ( ở đây giả sử có một ô $a$ cùng hàng và một ô $a$ cùng cột với ô $a+1$ ) chứa hai ô này thì bảng vẫn còn hai ô không bằng nhau . Nếu biển đổi một ô khác trong hàng và cột chứa ba ô này hoặc biến đổi hai ô này cũng vậy . Suy ra không thể có bảng thỏa mãn. Tóm lại luôn có hai ô mà hiệu khác $0$

Lập luận này không đúng rồi, phản chứng là ta sẽ biến đổi ô nằm cùng hàng với $1$ ô $a$ và cùng cột với ô $a$ kia (ô đó khác ô $a+1$) thì $2$ ô $a$ đó chuyển thành $a+1$, làm cho $3$ ô đó bằng nhau.

Ta dễ chứng minh rằng luôn tồn tại $1$ bảng $2\times 2$ chứa duy nhất $1$ số $1$. Bất biến ở đây chính là hiệu giữa tổng của $2$ số nằm trên mỗi đường chéo modulo 3 nên hiệu đó không chia hết cho $3$. Suy ra dpcm




#653462 tìm đa thức thỏa $\mathcal{P}(p)\mid 2^p-p$

Đã gửi bởi JUV on 09-09-2016 - 12:32 trong Đa thức

Có $\mathcal{P}(2)\mid 2 $,$\mathcal{P}(5)\mid 27 $ và $3\mid \mathcal{P}(5)- \mathcal{P}(2)$ nên hoặc $\mathcal{P}(2)=\mathcal{P}(5)=1$ hoặc $\mathcal{P}(2)=\mathcal{P}(5)=-1$.WOLG, giả sử $\mathcal{P}(2)=1$,ta thấy $\mathcal{P}(0)=1$ bởi nếu tồn tại $1$ ước nguyên tố lẻ của $\mathcal{P}(0)$ là $q$ thì $q\mid \mathcal{P}(q)$, hay $q\mid 2^q$ (vô lí). Xét $p$ nguyên tố bất kì, gọi $q$ là $1$ ước nguyên tố bất kì của $\mathcal{P}(p)$ thì $q\mid 2^p-p$, gọi $m= \text{ord}_{2}(q)$,giả sử $m>2$ có $(m;q)=1$ nên theo định lí thặng dư trung hoa thì $\left\{\begin{matrix} x\equiv p \pmod{q} & & \\ x\equiv p+v \pmod{m} & & \end{matrix}\right.$ với $v$ là $1$ số thỏa mãn $v$ không chia hết cho $m$ và $(p+v;m)=1$ (hoàn toàn chọn được do $m>2$ nên $\phi(m)\geq 2$), có $1$ nghiệm $x \pmod{qm}$ . Dễ thấy do $\mathcal{P}(0)=1$ nên $(p;q)=1$ và $(p+v;m)=1$ nên $(x;qm)=1$. Theo định lí Dirichlet, tồn tại $1$ số nguyên tố $t$ để $t \equiv x \pmod{qm}$. Có $q\mid t-p$ nên $q\mid \mathcal{P}(t)-\mathcal{P}(q)$$\Rightarrow$ $q\mid \mathcal{P}(t)$$\Rightarrow$$q\mid 2^t-t$$\Rightarrow$$q\mid 2^p(2^{t-p}-1)+p-t$$\Rightarrow$$q \mid 2^{t-p}-1$ (vô lí do $t-p$ không phải là bội của $m$). Vậy $m=2$, hay $q=3$, vậy $\mathcal{P}(p)$ có dạng $3^t$ với mọi $p$ nguyên tố(1). Cố định $p\neq 3$ và $t$, theo định lí Dirichlet thì tồn tại vô hạn số nguyên tố $q$ thỏa mãn $q\equiv p \pmod{3^{t+1}}$. Có $3^{t+1}\mid q-p$ nên $3^{t+1}\mid \mathcal{P}(q)-\mathcal{P}(p)$. Kết hợp với (1) ta có $\mathcal{P}(q)=3^t$.Vậy $\mathcal{P}(x)=3^t$ có vô số nghiệm nên đa thức là đa thức hằng.Thử lại thấy $\mathcal{P}(x)=1$ hoặc $\mathcal{P}(x)=-1$ thỏa mãn




#644561 Thảo luận về Đề thi và Lời giải của IMO 2016

Đã gửi bởi JUV on 11-07-2016 - 20:19 trong Thi HSG Quốc gia và Quốc tế

Có vẻ tình hình đội tuyển VN không ổn rồi. Tổ hợp là điểm yếu của đoàn mà ngay ngày đầu tiên đã phang 2 câu khó chơi rồi




#644742 Thảo luận về Đề thi và Lời giải của IMO 2016

Đã gửi bởi JUV on 13-07-2016 - 06:51 trong Thi HSG Quốc gia và Quốc tế

Bài 6:

a/ Xét 1 đoạn thẳng $a$ bất kì và $n-1$ điểm $A_{1};A_{2};...;A_{n-1}$ nằm trên nó, ta sẽ chọn $1$ đầu mút của nó và đánh các số $1;0$ theo thứ tự từ điểm gần chỗ đánh dấu nhất đến điểm xa nhất. Ta chọn đầu mút của đoạn thẳng đang xét ở phía $A_1$. Gọi $d_1;d_2;...;d_{n-1}$ là $n-1$ đoạn thẳng lần lượt đi qua $A_1;A_2;...A_{n-1}$ và lưu ý rằng nếu trong $1$ cách chọn đầu mút của đoạn thẳng $d$ mà điểm $A$ mang số $0$ thì với cách chọn còn lại thì $A$ sẽ mang số $1$ do $n$ lẻ, lúc này ta sẽ chọn đầu mút $n-1$ đoạn thẳng sao cho với $A_{i}$ được đánh số $1$ trên $a$ thì nó sẽ được đánh số $0$ trên $d_i$. Lúc này xét $2$ đoạn thẳng $d_i;d_j$ bất kì. Nếu $A_i$ và $A_j$ đánh số khác nhau trên $a$ thì nó cũng được đánh số khác nhau trên $d_i;d_j$ và có chẵn điểm nằm giữa $A_i$ và $A_j$ trên $a$. Xét $B$ là giao của $d_i;d_j$, lúc này ta thấy rằng nếu $1$ đường thẳng cắt $a$ tại điểm nằm ngoài $A_{i}A_{j}$ thì hoancj đường thẳng đó không cắt $BA_i$ và $BA_j$ hoạc không cắt đoạn nào. Còn $1$ đường thẳng cắt $a$ tại điểm ở giữa $A_i$ và $A_j$ thì nó sẽ cắt $1$ trong $2$ đoạn. Vì có chẵn điểm ở giữa $A_i$ và $A_j$ nên số điểm nằm trên đoạn $BA_i$ và $BA_j$ cùng tính chẵn lẻ. Mà $A_i$ và $A_j$ được đánh số khác nhau trên $d_i;d_j$ nên $B$ cũng được đánh số khác nhau trên $d_i;d_j$.Tương tự trường hợp giữa $A_i;A_j$ có chẵn điểm, chứng minh được giao điểm $2$ đoạn bất kì được đánh số khác nhau trên $2$ đoạn. Mà để con ếch tiến tới số $1$ trên $1$ đoạn thẳng thì cần lẻ lần thổi còi, số $0$ cần chẵn lần. Vì vậy $2$ con ếch không thể gặp nhau

b/(AOPS)Kéo dài $n$ đường thẳng thành $n$ đường thẳng, xét $1$ hình tròn chứa bên trong nó tất cả các giao điểm, hình tròn này cắt $n$ đường thẳng đó tại $2n$ điểm $P_1;P_2;...;P_{2n}$ (Xếp theo chiều KDH). Ta thấy rằng $P_i$ và $P_{i+n}$ cùng nằm trên $1$ đường thẳng với $i\leq n$ và nếu ta chọn $2$ đầu mút là $2$ điểm $P_i$ và $P_{i+1}$ thì $2$ con ếch ở $2$ đầu mút đó sẽ gặp nhau Vì vậy ta chỉ có thể chọn các đầu mút là $P_1;P_3;...;P_{2n-1}$ tuy nhiên khi đó $2$ đầu mút cạnh nhau là $P_{n+1}$ và $P_1$ nên $2$ con ếch ở $2$ đầu mút đó sẽ gặp nhau, tương tự với trường hợp chọn $P_2;P_4;...P_{2n}$, ta đều không thể chọn đầu mút sao cho không có $2$ con ếch nào gặp nhau




#644550 Thảo luận về Đề thi và Lời giải của IMO 2016

Đã gửi bởi JUV on 11-07-2016 - 19:14 trong Thi HSG Quốc gia và Quốc tế

Bài 2: Dễ thấy $n\vdots 3$, đặt $n=3t$, chia bảng $n\times n$ thành $t^2$ bảng con $3\times 3$ thì lúc này xét $t$ hàng và $t$ cột sao cho mỗi hàng và cột đó đều đi qua ô chính giữa của $t$ bảng $3\times 3$, ta cũng xét $4n-2$ đường chéo như đề bài cho, thấy rằng mỗi đường chéo này đều đi qua ô chính giữa của $1$ số bảng $3\times 3$ .Gọi $t$ hàng, $t$ cột và $4n-2$ đường chéo đã xét là các đường, mỗi ô chính giữa của các bảng $3\times 3$ đều có đúng $3$ đường đi qua và các ô khác chỉ có đúng $1$ đường đi qua. Mà theo đầu bài thì mỗi đường chứa các chữ $I,M,O$ bằng nhau và cũng dễ nhận thấy rằng số các chữ cái $I,M,O$ viết trên bảng cũng bằng nhau nên nếu xét $t^2$ ô là ô chính giữa của $t^2$ bảng thì số các ô được viết chữ $I,M,O$ là bằng nhau. Vì vậy $t^2\vdots 3$ nên $n\vdots 9$. Xét trường hợp $n=9$ có thể dễ dàng chỉ ra $1$ bảng thoả mãn. Còn với $n=9k$ với $k\in \mathbb{N}$ bất kì thì chỉ cần ghép $k^2$ bảng $9\times 9$ thoả mãn đề bài với nhau để tạo được $1$ bảng $9k\times 9k$ thoả mãn đề bài




#644595 Thảo luận về Đề thi và Lời giải của IMO 2016

Đã gửi bởi JUV on 11-07-2016 - 23:03 trong Thi HSG Quốc gia và Quốc tế

Không biết có phải trùng hợp hay không nhưng bài 2 có cấu hình "khá giống" với bài 4 VMO 2016 :D

 

http://diendantoanho...-giải-vmo-2016/

Mình có thấy trùng tí nào đâu. Bài VMO thì lời giải trâu bò hơn nhưng bài IMO mới là bài khó hơn